Evaluate the line integral $int_C xy^4 ds $ of a half circle












0












$begingroup$



Evaluate the line integral where $C$ is the given curve:
$int_C xy^4 ds $, $C$ it the right half of the circle $x^2+y^2=16$




enter image description here



I was following a similar example in my book and parameterized $t$ by making $x = cos t$ and $y = sin t$ and using the line integral equation, but I got a small fraction while the answer is a large number. I looked at the example twice and checked my problem twice, so I'm not sure what I'm missing. If anyone could help it would be greatly appreciated..










share|cite|improve this question











$endgroup$








  • 2




    $begingroup$
    the correct parameterization is x=4*cos(t), y=4*sin(t)
    $endgroup$
    – gimusi
    Nov 26 '17 at 18:39










  • $begingroup$
    I actually was looking at my problem again and just realized that lol
    $endgroup$
    – 2316354654
    Nov 26 '17 at 18:40










  • $begingroup$
    alright I plugged in the right parametric values and my radical came out to be 1/4 and the whole thing came out to be 512/5 which is 102.4.. but the right answer is way bigger...???
    $endgroup$
    – 2316354654
    Nov 26 '17 at 18:43










  • $begingroup$
    you have to put also a $rho$ to take into account the variables change
    $endgroup$
    – gimusi
    Nov 26 '17 at 18:45






  • 1




    $begingroup$
    If you want someone to point out where you’re going wrong, you should include your work in the question instead of simply writing that you keep getting the wrong answer.
    $endgroup$
    – amd
    Nov 26 '17 at 23:27
















0












$begingroup$



Evaluate the line integral where $C$ is the given curve:
$int_C xy^4 ds $, $C$ it the right half of the circle $x^2+y^2=16$




enter image description here



I was following a similar example in my book and parameterized $t$ by making $x = cos t$ and $y = sin t$ and using the line integral equation, but I got a small fraction while the answer is a large number. I looked at the example twice and checked my problem twice, so I'm not sure what I'm missing. If anyone could help it would be greatly appreciated..










share|cite|improve this question











$endgroup$








  • 2




    $begingroup$
    the correct parameterization is x=4*cos(t), y=4*sin(t)
    $endgroup$
    – gimusi
    Nov 26 '17 at 18:39










  • $begingroup$
    I actually was looking at my problem again and just realized that lol
    $endgroup$
    – 2316354654
    Nov 26 '17 at 18:40










  • $begingroup$
    alright I plugged in the right parametric values and my radical came out to be 1/4 and the whole thing came out to be 512/5 which is 102.4.. but the right answer is way bigger...???
    $endgroup$
    – 2316354654
    Nov 26 '17 at 18:43










  • $begingroup$
    you have to put also a $rho$ to take into account the variables change
    $endgroup$
    – gimusi
    Nov 26 '17 at 18:45






  • 1




    $begingroup$
    If you want someone to point out where you’re going wrong, you should include your work in the question instead of simply writing that you keep getting the wrong answer.
    $endgroup$
    – amd
    Nov 26 '17 at 23:27














0












0








0





$begingroup$



Evaluate the line integral where $C$ is the given curve:
$int_C xy^4 ds $, $C$ it the right half of the circle $x^2+y^2=16$




enter image description here



I was following a similar example in my book and parameterized $t$ by making $x = cos t$ and $y = sin t$ and using the line integral equation, but I got a small fraction while the answer is a large number. I looked at the example twice and checked my problem twice, so I'm not sure what I'm missing. If anyone could help it would be greatly appreciated..










share|cite|improve this question











$endgroup$





Evaluate the line integral where $C$ is the given curve:
$int_C xy^4 ds $, $C$ it the right half of the circle $x^2+y^2=16$




enter image description here



I was following a similar example in my book and parameterized $t$ by making $x = cos t$ and $y = sin t$ and using the line integral equation, but I got a small fraction while the answer is a large number. I looked at the example twice and checked my problem twice, so I'm not sure what I'm missing. If anyone could help it would be greatly appreciated..







calculus integration multivariable-calculus parametrization






share|cite|improve this question















share|cite|improve this question













share|cite|improve this question




share|cite|improve this question








edited Dec 12 '18 at 2:12









Martin Sleziak

44.7k10119272




44.7k10119272










asked Nov 26 '17 at 18:35









23163546542316354654

306213




306213








  • 2




    $begingroup$
    the correct parameterization is x=4*cos(t), y=4*sin(t)
    $endgroup$
    – gimusi
    Nov 26 '17 at 18:39










  • $begingroup$
    I actually was looking at my problem again and just realized that lol
    $endgroup$
    – 2316354654
    Nov 26 '17 at 18:40










  • $begingroup$
    alright I plugged in the right parametric values and my radical came out to be 1/4 and the whole thing came out to be 512/5 which is 102.4.. but the right answer is way bigger...???
    $endgroup$
    – 2316354654
    Nov 26 '17 at 18:43










  • $begingroup$
    you have to put also a $rho$ to take into account the variables change
    $endgroup$
    – gimusi
    Nov 26 '17 at 18:45






  • 1




    $begingroup$
    If you want someone to point out where you’re going wrong, you should include your work in the question instead of simply writing that you keep getting the wrong answer.
    $endgroup$
    – amd
    Nov 26 '17 at 23:27














  • 2




    $begingroup$
    the correct parameterization is x=4*cos(t), y=4*sin(t)
    $endgroup$
    – gimusi
    Nov 26 '17 at 18:39










  • $begingroup$
    I actually was looking at my problem again and just realized that lol
    $endgroup$
    – 2316354654
    Nov 26 '17 at 18:40










  • $begingroup$
    alright I plugged in the right parametric values and my radical came out to be 1/4 and the whole thing came out to be 512/5 which is 102.4.. but the right answer is way bigger...???
    $endgroup$
    – 2316354654
    Nov 26 '17 at 18:43










  • $begingroup$
    you have to put also a $rho$ to take into account the variables change
    $endgroup$
    – gimusi
    Nov 26 '17 at 18:45






  • 1




    $begingroup$
    If you want someone to point out where you’re going wrong, you should include your work in the question instead of simply writing that you keep getting the wrong answer.
    $endgroup$
    – amd
    Nov 26 '17 at 23:27








2




2




$begingroup$
the correct parameterization is x=4*cos(t), y=4*sin(t)
$endgroup$
– gimusi
Nov 26 '17 at 18:39




$begingroup$
the correct parameterization is x=4*cos(t), y=4*sin(t)
$endgroup$
– gimusi
Nov 26 '17 at 18:39












$begingroup$
I actually was looking at my problem again and just realized that lol
$endgroup$
– 2316354654
Nov 26 '17 at 18:40




$begingroup$
I actually was looking at my problem again and just realized that lol
$endgroup$
– 2316354654
Nov 26 '17 at 18:40












$begingroup$
alright I plugged in the right parametric values and my radical came out to be 1/4 and the whole thing came out to be 512/5 which is 102.4.. but the right answer is way bigger...???
$endgroup$
– 2316354654
Nov 26 '17 at 18:43




$begingroup$
alright I plugged in the right parametric values and my radical came out to be 1/4 and the whole thing came out to be 512/5 which is 102.4.. but the right answer is way bigger...???
$endgroup$
– 2316354654
Nov 26 '17 at 18:43












$begingroup$
you have to put also a $rho$ to take into account the variables change
$endgroup$
– gimusi
Nov 26 '17 at 18:45




$begingroup$
you have to put also a $rho$ to take into account the variables change
$endgroup$
– gimusi
Nov 26 '17 at 18:45




1




1




$begingroup$
If you want someone to point out where you’re going wrong, you should include your work in the question instead of simply writing that you keep getting the wrong answer.
$endgroup$
– amd
Nov 26 '17 at 23:27




$begingroup$
If you want someone to point out where you’re going wrong, you should include your work in the question instead of simply writing that you keep getting the wrong answer.
$endgroup$
– amd
Nov 26 '17 at 23:27










3 Answers
3






active

oldest

votes


















0












$begingroup$

$$r(t)=(4cos t,,4sin t);,;;-fracpi2le tlefracpi2implies text{our integral is}$$



$$int_{-fracpi2}^{fracpi2}4cos t(4sin t)^4left|r'(t)right|dt=1,024int_{-fracpi2}^{fracpi2}cos t,sin^4tcdot4,dt=left.frac{4,096}5sin^5tright|_{-fracpi2}^{fracpi2}=frac{8,192}5$$






share|cite|improve this answer









$endgroup$













  • $begingroup$
    how did you get the 4 for the norm of r'(t) ?
    $endgroup$
    – 2316354654
    Nov 26 '17 at 18:44










  • $begingroup$
    why do you use commata?
    $endgroup$
    – klirk
    Nov 26 '17 at 18:47










  • $begingroup$
    wait nevermind i see it.
    $endgroup$
    – 2316354654
    Nov 26 '17 at 18:47



















1












$begingroup$

You forgot to include the radius. The substitution $x= cos t$, $y= sin t$ parametrizes a circle of radius $1$.

But $C$ is a subset of a circle of radius $4$.






share|cite|improve this answer









$endgroup$





















    0












    $begingroup$

    The correct parameterization is $x=4cos(t), y=4sin(t)$.






    share|cite|improve this answer











    $endgroup$













      Your Answer





      StackExchange.ifUsing("editor", function () {
      return StackExchange.using("mathjaxEditing", function () {
      StackExchange.MarkdownEditor.creationCallbacks.add(function (editor, postfix) {
      StackExchange.mathjaxEditing.prepareWmdForMathJax(editor, postfix, [["$", "$"], ["\\(","\\)"]]);
      });
      });
      }, "mathjax-editing");

      StackExchange.ready(function() {
      var channelOptions = {
      tags: "".split(" "),
      id: "69"
      };
      initTagRenderer("".split(" "), "".split(" "), channelOptions);

      StackExchange.using("externalEditor", function() {
      // Have to fire editor after snippets, if snippets enabled
      if (StackExchange.settings.snippets.snippetsEnabled) {
      StackExchange.using("snippets", function() {
      createEditor();
      });
      }
      else {
      createEditor();
      }
      });

      function createEditor() {
      StackExchange.prepareEditor({
      heartbeatType: 'answer',
      autoActivateHeartbeat: false,
      convertImagesToLinks: true,
      noModals: true,
      showLowRepImageUploadWarning: true,
      reputationToPostImages: 10,
      bindNavPrevention: true,
      postfix: "",
      imageUploader: {
      brandingHtml: "Powered by u003ca class="icon-imgur-white" href="https://imgur.com/"u003eu003c/au003e",
      contentPolicyHtml: "User contributions licensed under u003ca href="https://creativecommons.org/licenses/by-sa/3.0/"u003ecc by-sa 3.0 with attribution requiredu003c/au003e u003ca href="https://stackoverflow.com/legal/content-policy"u003e(content policy)u003c/au003e",
      allowUrls: true
      },
      noCode: true, onDemand: true,
      discardSelector: ".discard-answer"
      ,immediatelyShowMarkdownHelp:true
      });


      }
      });














      draft saved

      draft discarded


















      StackExchange.ready(
      function () {
      StackExchange.openid.initPostLogin('.new-post-login', 'https%3a%2f%2fmath.stackexchange.com%2fquestions%2f2538372%2fevaluate-the-line-integral-int-c-xy4-ds-of-a-half-circle%23new-answer', 'question_page');
      }
      );

      Post as a guest















      Required, but never shown

























      3 Answers
      3






      active

      oldest

      votes








      3 Answers
      3






      active

      oldest

      votes









      active

      oldest

      votes






      active

      oldest

      votes









      0












      $begingroup$

      $$r(t)=(4cos t,,4sin t);,;;-fracpi2le tlefracpi2implies text{our integral is}$$



      $$int_{-fracpi2}^{fracpi2}4cos t(4sin t)^4left|r'(t)right|dt=1,024int_{-fracpi2}^{fracpi2}cos t,sin^4tcdot4,dt=left.frac{4,096}5sin^5tright|_{-fracpi2}^{fracpi2}=frac{8,192}5$$






      share|cite|improve this answer









      $endgroup$













      • $begingroup$
        how did you get the 4 for the norm of r'(t) ?
        $endgroup$
        – 2316354654
        Nov 26 '17 at 18:44










      • $begingroup$
        why do you use commata?
        $endgroup$
        – klirk
        Nov 26 '17 at 18:47










      • $begingroup$
        wait nevermind i see it.
        $endgroup$
        – 2316354654
        Nov 26 '17 at 18:47
















      0












      $begingroup$

      $$r(t)=(4cos t,,4sin t);,;;-fracpi2le tlefracpi2implies text{our integral is}$$



      $$int_{-fracpi2}^{fracpi2}4cos t(4sin t)^4left|r'(t)right|dt=1,024int_{-fracpi2}^{fracpi2}cos t,sin^4tcdot4,dt=left.frac{4,096}5sin^5tright|_{-fracpi2}^{fracpi2}=frac{8,192}5$$






      share|cite|improve this answer









      $endgroup$













      • $begingroup$
        how did you get the 4 for the norm of r'(t) ?
        $endgroup$
        – 2316354654
        Nov 26 '17 at 18:44










      • $begingroup$
        why do you use commata?
        $endgroup$
        – klirk
        Nov 26 '17 at 18:47










      • $begingroup$
        wait nevermind i see it.
        $endgroup$
        – 2316354654
        Nov 26 '17 at 18:47














      0












      0








      0





      $begingroup$

      $$r(t)=(4cos t,,4sin t);,;;-fracpi2le tlefracpi2implies text{our integral is}$$



      $$int_{-fracpi2}^{fracpi2}4cos t(4sin t)^4left|r'(t)right|dt=1,024int_{-fracpi2}^{fracpi2}cos t,sin^4tcdot4,dt=left.frac{4,096}5sin^5tright|_{-fracpi2}^{fracpi2}=frac{8,192}5$$






      share|cite|improve this answer









      $endgroup$



      $$r(t)=(4cos t,,4sin t);,;;-fracpi2le tlefracpi2implies text{our integral is}$$



      $$int_{-fracpi2}^{fracpi2}4cos t(4sin t)^4left|r'(t)right|dt=1,024int_{-fracpi2}^{fracpi2}cos t,sin^4tcdot4,dt=left.frac{4,096}5sin^5tright|_{-fracpi2}^{fracpi2}=frac{8,192}5$$







      share|cite|improve this answer












      share|cite|improve this answer



      share|cite|improve this answer










      answered Nov 26 '17 at 18:42









      DonAntonioDonAntonio

      179k1494230




      179k1494230












      • $begingroup$
        how did you get the 4 for the norm of r'(t) ?
        $endgroup$
        – 2316354654
        Nov 26 '17 at 18:44










      • $begingroup$
        why do you use commata?
        $endgroup$
        – klirk
        Nov 26 '17 at 18:47










      • $begingroup$
        wait nevermind i see it.
        $endgroup$
        – 2316354654
        Nov 26 '17 at 18:47


















      • $begingroup$
        how did you get the 4 for the norm of r'(t) ?
        $endgroup$
        – 2316354654
        Nov 26 '17 at 18:44










      • $begingroup$
        why do you use commata?
        $endgroup$
        – klirk
        Nov 26 '17 at 18:47










      • $begingroup$
        wait nevermind i see it.
        $endgroup$
        – 2316354654
        Nov 26 '17 at 18:47
















      $begingroup$
      how did you get the 4 for the norm of r'(t) ?
      $endgroup$
      – 2316354654
      Nov 26 '17 at 18:44




      $begingroup$
      how did you get the 4 for the norm of r'(t) ?
      $endgroup$
      – 2316354654
      Nov 26 '17 at 18:44












      $begingroup$
      why do you use commata?
      $endgroup$
      – klirk
      Nov 26 '17 at 18:47




      $begingroup$
      why do you use commata?
      $endgroup$
      – klirk
      Nov 26 '17 at 18:47












      $begingroup$
      wait nevermind i see it.
      $endgroup$
      – 2316354654
      Nov 26 '17 at 18:47




      $begingroup$
      wait nevermind i see it.
      $endgroup$
      – 2316354654
      Nov 26 '17 at 18:47











      1












      $begingroup$

      You forgot to include the radius. The substitution $x= cos t$, $y= sin t$ parametrizes a circle of radius $1$.

      But $C$ is a subset of a circle of radius $4$.






      share|cite|improve this answer









      $endgroup$


















        1












        $begingroup$

        You forgot to include the radius. The substitution $x= cos t$, $y= sin t$ parametrizes a circle of radius $1$.

        But $C$ is a subset of a circle of radius $4$.






        share|cite|improve this answer









        $endgroup$
















          1












          1








          1





          $begingroup$

          You forgot to include the radius. The substitution $x= cos t$, $y= sin t$ parametrizes a circle of radius $1$.

          But $C$ is a subset of a circle of radius $4$.






          share|cite|improve this answer









          $endgroup$



          You forgot to include the radius. The substitution $x= cos t$, $y= sin t$ parametrizes a circle of radius $1$.

          But $C$ is a subset of a circle of radius $4$.







          share|cite|improve this answer












          share|cite|improve this answer



          share|cite|improve this answer










          answered Nov 26 '17 at 18:40









          klirkklirk

          2,788630




          2,788630























              0












              $begingroup$

              The correct parameterization is $x=4cos(t), y=4sin(t)$.






              share|cite|improve this answer











              $endgroup$


















                0












                $begingroup$

                The correct parameterization is $x=4cos(t), y=4sin(t)$.






                share|cite|improve this answer











                $endgroup$
















                  0












                  0








                  0





                  $begingroup$

                  The correct parameterization is $x=4cos(t), y=4sin(t)$.






                  share|cite|improve this answer











                  $endgroup$



                  The correct parameterization is $x=4cos(t), y=4sin(t)$.







                  share|cite|improve this answer














                  share|cite|improve this answer



                  share|cite|improve this answer








                  edited Dec 11 '18 at 14:19









                  Davide Giraudo

                  127k16151264




                  127k16151264










                  answered Nov 26 '17 at 18:40









                  gimusigimusi

                  92.9k84494




                  92.9k84494






























                      draft saved

                      draft discarded




















































                      Thanks for contributing an answer to Mathematics Stack Exchange!


                      • Please be sure to answer the question. Provide details and share your research!

                      But avoid



                      • Asking for help, clarification, or responding to other answers.

                      • Making statements based on opinion; back them up with references or personal experience.


                      Use MathJax to format equations. MathJax reference.


                      To learn more, see our tips on writing great answers.




                      draft saved


                      draft discarded














                      StackExchange.ready(
                      function () {
                      StackExchange.openid.initPostLogin('.new-post-login', 'https%3a%2f%2fmath.stackexchange.com%2fquestions%2f2538372%2fevaluate-the-line-integral-int-c-xy4-ds-of-a-half-circle%23new-answer', 'question_page');
                      }
                      );

                      Post as a guest















                      Required, but never shown





















































                      Required, but never shown














                      Required, but never shown












                      Required, but never shown







                      Required, but never shown

































                      Required, but never shown














                      Required, but never shown












                      Required, but never shown







                      Required, but never shown







                      Popular posts from this blog

                      Bundesstraße 106

                      Le Mesnil-Réaume

                      Ida-Boy-Ed-Garten